Monica has a yearly salary of $29,700. Her employer withholds $2948 in state and federal taxes and $2573 in FICA taxes throughout the year. She has the following monthly costs: transportation is $140, cell phone bill is $65, student loans require $300 in repayment, and rent is $675. She is using the average monthly costs for each of the following in order to gain an idea of other monthly expenses: utilities are $130, internet is $55, health insurance is $278, and groceries are $220. What is Monica’s monthly net pay amount? Round your answer to the nearest cent, if necessary.

Answers

Answer 1

Step-by-step explanation:

your teacher is not very precise, I fear.

the yearly salary ($29,700) is the gross pay she gets.

the gross pay minus the taxes ($2,948 and $2,573) is then the net pay she receives.

in many cases the employer also withholds the health insurance premium (and then that has to be subtracted too before getting the net pay).

I just think your teacher means by "net pay" how much she will have left are all the monthly costs.

her annual net pay amount is

$29,700 - $2,948 - $2,573 = $24,179

the monthly net pay is $24,179/12 = $2,014.916666... ≈

≈ $2,014.92

she has to pay monthly

$140

$ 65

$300

$675

$130

$ 55

$278

$220

---------

$1,863

based on her monthly net pay she has left

2,014.916666...

- 1,863

---------------------------

151.9166667 ≈ $151.92

per month

please pick based on what you think your teacher means by net pay : the actual net pay by her employer, or how much money she has monthly left after paying all bills.


Related Questions

ERROR ANALYSIS: Student A says there is no solution to the graphed system of equations. Student B says there is one solution. Which student is correct? Why?

Answers

Then, to solve the exercise, we need to know the slope of each line. The slope of a line is the measure of the steepness of a line.

The graph shows that the red line has a greater steepness than the blue line. Thus, the lines have different slopes. Also, two lines are parallel if they have the same slope. Since these lines have different slopes, they are not parallel.

Therefore, student B is correct because the lines are not parallel, so they will intersect.

Enter the ratio as a fraction in lowest terms
2ft to 40 in.

Answers

Answer:

[tex]\frac{3}{5}[/tex]

Step-by-step explanation:

2 feet is equivalent to 24 inches.  (12 inches make a foot 2 x 12 = 24)

[tex]\frac{24}{40}[/tex] = [tex]\frac{3}{5}[/tex]  Divide the top (numerator) and the bottom (denominator) by 8 to simplify.

Solve: 12/40 = 30/×

Answers

Given:

[tex]\frac{12}{40}=\frac{30}{x}[/tex][tex]\begin{gathered} x=30\times\frac{40}{12} \\ x=\frac{1200}{12} \\ x=100 \end{gathered}[/tex]

Answer:

100

Step-by-step explanation:

Solve:

12/40 = 30/×

it's a simple equation

12 : 40 = 30 : x

x = 30 * 40 : 12

x = 1200 : 12

x = 100

to convert a decimal to do a fraction for the decimal over ______and multiply the numerator and denominator by _______and tell there is no longer a decimal in the numerator then___ your fraction

Answers

1) Let's proceed the steps

To convert a decimal to do a fraction for the decimal over one

Ex

0.5/1

And multiply the numerator and denominator by ten

Ex

5/10

(..) *until there is no longer a decimal in the numerator then simplify your fraction

Ex

1/2

a casting weighted 148 lb out of the mold. it weighed 141 lb after finishing. what percent of the weight was lost in the finishing?

Answers

The amount of weight lost will be equal to 4.72%.

Percentage may be defined as a form of expressing a number as a fraction of hundred. To find the percentage of a number we divide the number by the total amount and then multiply the answer with hundred. The initial weight of the casting was 148 lb and the final weight is 141 lb. To find the percentage we use the formula

Percentage lost = [(Final weight - Initial weight)/Initial weight] × 100

Percentage lost = [(141 - 148)/148] × 100

Percentage lost = ( -7/148) × 100

Percentage lost = -0.0472 × 100

Percentage lost = -4.72% and since we write the absolute value therefore, Percentage lost = 4.72%.

Learn more about percentage at:

brainly.com/question/26352729

#SPJ9

What is the slope of a line perpendicular to the line whose equation is 6x+8y=−128. Fully simplify your answer.

Answers

The slope of a line perpendicular to the line 6x + 8y = -128 is 4/3.

Given,

The equation of a line = 6x + 8y = -128

We have to convert this into standard form of linear equation, y = mx + b :

So,

6x + 8y = -128

Add -6x to both sides,

6x + 8y - 6x = -6x - 128

Now,

8y = -6x - 128

Divide 8 on both sides

8y/8 = (-6x - 128) / 8

We get,

y = -6/8x - 16

Here, this is in standard form of linear equation.

Here slope of line, m₁ = -6/8

We have to find the slope of line(m₂) which is perpendicular to the given line.

If the line is perpendicular, the slope(m₂) will be the negative reciprocal of the slope(m₁) of the given line.

That is,

Slope of line, (m₂)= -(m₁) =  - (-8/6) = 8/6 = 4/3

That is, the slope of the line which is perpendicular to the given line is 4/3.

Learn more about slope of the line here:

https://brainly.com/question/16616984

#SPJ1

Simplify using order of
operations.
3 • 10 (9 + 1)²

Answers

Answer:

3000

Step-by-step explanation:

Parentheses - (9 + 1) = 10

Exponents - 10 x 10 = 100

Multiplication - 10 x 100 = 1000 then 1000 x 3 = 3000

D

A

S

Write an expression that is the product of two factors and is equivalent to - 3x - 15.Which expression is the product of two factors and is equivalent to - 3x - 15?A. - 3x-2-13B.-3(x - 5)C. (7x-2)-(10x - 13)D. -3(x+5)

Answers

Amon the choices given, letter D is equivalent to - 3x - 15.

-3 ( x + 5 )

if we distribute -3 , we will have

-3 (x ) = -3x , and

-3 ( +5) = -15

-3 ( x + 5) = -3x - 15

Answer: D. -3 (X + 5)

Real world compositionsWhat is the volume of a spherical balloon after 11 seconds if the radius of the balloon is increasing at 1.3 cm/sec? Round to the nearest tenth of a centimeter solve using composite functions

Answers

In this case we have two functions. The first one is the volume of the balloon and the other is the value of the radius. Both functions depend on the variable time.

Finding the functions to solve the problem, we have:

[tex]\begin{gathered} V(t)=\frac{4}{3}\pi r^3\text{ }^{}\text{ (First function)} \\ r(t)=1.3\cdot t\text{(Second function. Let us suppose that the initial radius is equal to zero)} \end{gathered}[/tex]

Let us replace the second function in the first one to get a composite function. Doing so, we have:

[tex]\begin{gathered} V(t)=\frac{4}{3}\pi(1.3\cdot t)^3 \\ V(t)=\frac{4}{3}\pi(2.197\cdot t^3)\text{ (Raising the expression within parentheses to the power of 3)} \\ V(t)=2.93\pi\cdot t^3(^{}\text{ Multiplying constant terms)} \end{gathered}[/tex]

Now, we can replace t=11 to find the value of the volume.

[tex]\begin{gathered} V(11)=2.93\pi\cdot(11)^3\text{ } \\ V(11)=2.93\pi(1331)\text{ (Raising 11 to the power of 3)} \\ V(11)=12248.88\text{ (Multiplying)} \end{gathered}[/tex]

The answer is 12248.9 cm3 (Rounding to the nearest tenth of a centimeter).

Could you help me with these problems ? I don't really know how to put the equation together and the steps I should take to solve it, it would be amazing if you could show me how, and I'd be very thankful if you could also add a picture showing the steps if possible. Thank you so much.

Answers

Since it is a right triangle, you can use the trigonometric ratio sin(θ) to solve the exercise:

[tex]\sin (\theta)=\frac{\text{opposite side}}{\text{hypotenuse}}[/tex]

Graphically,

So, in this case, you have

[tex]\begin{gathered} \theta=54\text{\degree} \\ \text{ Opposite side }=72 \\ \text{ Hypotenuse }=x \\ \sin (54\text{\degree})=\frac{72}{x} \\ \text{ Multiply by x from both sides of the equation} \\ \sin (54\text{\degree})\cdot x=\frac{72}{x}\cdot x \\ \sin (54\text{\degree})\cdot x=72 \\ \text{ Divide by }\sin (54\text{\degree})\text{ from both sides of the equation} \\ \frac{\sin(54\text{\degree})\cdot x}{\sin(54\text{\degree})}=\frac{72}{\sin(54\text{\degree})} \\ x=\frac{72}{\sin(54\text{\degree})} \\ x=\frac{72}{0.8090} \\ x=88.99 \\ \text{ Rounding to the nearest tenth} \\ x=89.0 \end{gathered}[/tex]

Therefore, the measure of the missing side is 89.

Does the point (1, 0) satisfy the equation y = 7x + 1? yes no

Answers

To find out if the point (1,0) satisfy the equation we plug x=1 and y=0 into the equation:

[tex]\begin{gathered} 0=7\cdot1+1 \\ 0=8 \end{gathered}[/tex]

since this is not true we conclude that th


Determine whether the statement makes sense or does not make sense, and explain your reasoning.
.I read that a certain star is 10^5 light-years from Earth, which is 100,000 light-years.
Choose the correct answer below.
OA. The statement makes sense. The value of 10^5 is not 100,000.
OB. The statement makes sense. The value of 10^5 is 100,000.
OC. The statement does not make sense. The value of 10^5 is not 100,000.
OD. The statement does not make sense. The value of 10^5 is 100,000.

Answers

OB. The statement makes sense. The value of 10^5 is 100,000 is the correct answer.

How to expand exponential expressions?

10^5 = 10*10*10*10*10 = 100,000

Repeated multiplication is expressed using exponential notation.

For instance, 10*10*10 can be expressed more briefly as 10^3.

The base is the number 10 in 10^3.

Exponent refers to the 3 in 10^3.

The exponential expression is known as the number 10^3.

You can learn how to conduct mathematical operations on an exponential expression or term by being familiar with its constituent parts' names.

To learn more about exponential expression, refer

https://brainly.com/question/8844911

#SPJ13

Your friend just purchased a new sports car for $32,000. He received $6,000 for his trade in and he used that money as a down payment for the new sports car. He financed the vehicle at 6.76% APR over 48 months. Determine the amount financed from the given information.
a.
$38,000
c.
$29746.15
b.
$32,000
d.
$26,000

Answers

Answer: C. $29,746.08

Step-by-step explanation:

Rationalise
(3+√2)/(√5+√3)

Answers

Rationalization of the surd [tex]\frac{(3+\sqrt{2} )}{(\sqrt{5} )+\sqrt{3} }[/tex] gives [tex]\frac{3\sqrt{5} -3\sqrt{3}+\sqrt{10} -\sqrt{6} }{2}[/tex]

What is surd?

In mathematics surd refers to the root of numbers that do not have a perfect root. Surds are used to represent irrational numbers in square root, cube root and so on.

How to Rationalize (3+√2)/(√5+√3)

The data given the question is

[tex]\frac{(3+\sqrt{2} )}{(\sqrt{5} )+\sqrt{3} }[/tex]

This is solved as follows;

[tex]\frac{(3+\sqrt{2} )}{(\sqrt{5} )+\sqrt{3} }[/tex]

Rationalizing the denominator

[tex]\frac{(3+\sqrt{2} )}{(\sqrt{5} +\sqrt{3} }*\frac{\sqrt{5} -\sqrt{3} }{\sqrt{5} -\sqrt{3} }[/tex]

multiplying out

[tex]\frac{3\sqrt{5}-3\sqrt{3} +\sqrt{10}-\sqrt{6} }{5-\sqrt{15} +\sqrt{15} -3}[/tex]

adding and subtraction when required

[tex]\frac{3\sqrt{5}-3\sqrt{3} +\sqrt{10}-\sqrt{6} }{5-3}[/tex]

[tex]\frac{3\sqrt{5}-3\sqrt{3} +\sqrt{10}-\sqrt{6} }{2}[/tex]

Learn more about rationalization of surds at : https://brainly.com/question/14261303

#SPJ1

A company is conducting a survey to determine how prepared people are for a long-term power outage, natural disaster, or terrorist attack. The frequency distribution on the right shows the resultsUse the table to answer the following question. What is the probability that the next person surveyed is very prepared?

Answers

The probability the next person surveyed is very prepared is 0.119.

What actually does probability mean?

The area of mathematics known as probability deals with numerical representations of the likelihood that an event will occur or that a statement is true. An event's probability is a number ranging from zero and 1, where, roughly, 0 denotes the event's impossibility and 1 represents certainty.

Formula of total frequency is,

                           Total = f₁ + f₂  + f₃ + f₄ +...............n∑fi

The total frequency is,

                      total = n∑i=2 fi ⇒ 262 + 992 + 587 + 313 + 52

The total number of people prepared for a long-term power outage, natural disaster, or terrorist attack is obtained by taking the sum of the all frequencies.

               probability = N(E)/N(S)

the information given, there are 262 people are very prepared.

That is,      N(A) = 262

The required probability is,

                            P(A) = 262/2206

                                     = 0.119

The probability the next person surveyed is very prepared is 0.119.

Learn more about probability

brainly.com/question/11234923

#SPJ13

David watches Maria and Alma race electric trains around a track. Maria's train goes around the track in 10 seconds. Alma's train goes around the track in 12 seconds.

How long will it take for both trains to cross the finish line together the first time?

Answers

Answer:

60 seconds

Step-by-step explanation:

The LCM of 10 and 12 is

10 = 2 × 5

12 = 2² × 3

LCM = 2² × 3 × 5 = 60

Answer: 60 seconds

the anwser is 60 seconds

You are cycling around Europe with your friends.You want to frame a photo from your trip to send home. Select a frame that is suitable for a photo with a perimeter of 70 cm. Is this frame suitable?YesNoIs this frame suitable?YesNoIs this frame suitable?YesNo

Answers

The perimeter of frame is given 70 cm.

There are two frames given.

For first frame , calculating the perimeter .

[tex]P=2(15+20)=2\times35=70\operatorname{cm}[/tex]

For second frame, calculating the perimeter.

[tex]P=2(30+40)=2\times70=140\operatorname{cm}[/tex]

Hence the first frame is suitable for for a photo with a perimeter of 70 cm.

This is the frame which is suitable.

Given g(k) = 2k² - 4, find g(-5).

Answers

Answer:

g(-5)=46

Step-by-step explanation:

given that function is,

g(k) = 2k² - 4,

and we find the value of g(-5)

so,

g(-5)=2(5)^2-4

simpley put -5 in place of k

g(-5)=2×25-4

g(-5)=46

Answer:

g(-5) = 46

Step-by-step explanation:

Given function:

[tex]g(k)=2k^2-4[/tex]

To find g(-5), substitute k = -5 into the given function:

[tex]\begin{aligned}\implies g(-5)&=2(-5)^2-4\\&=2(25)-4\\&=50-4\\&=46\end{aligned}[/tex]

$4500 is deposited for 4.5 years in an account that pays 4.5% interest compounded monthly. What is the value of the account when the customer takes the money at the end of the 4.5 years?

Answers

Answer:

$5507.98

Explanation:

To find the value of the account, we use the compound interest formula below:

[tex]Amount\: at\: Compound\: Interest,A=P(1+\frac{r}{n})^{nt}[/tex]

From the given information:

• Principal,P=$4500

,

• Interest Rate, r=4.5%=0.045

,

• Number of compounding periods, n=12 (Monthly)

,

• Time, t=4.5 years

Substituting the given values, we have:

[tex]\begin{gathered} A=4500(1+\frac{0.045}{12})^{12\times4.5} \\ =4500(1+0.00375)^{54} \\ =4500(1.00375)^{54} \\ =\$5507.98 \end{gathered}[/tex]

The value of the account when the customer takes the money at the end of the 4.5 years is $5507.98.

Steve Conway wants his team to win more than 57 games this year. His team has already won 2 games this season and there are 5 more months to play. Based on his goal, how many games must his team win per month?

Answers

Using a linear function, it is found that:

The inequality is: 5m + 2 > 57.The solution is: m > 11.They team needs to win more than 11 games per month.

Linear function

The linear function that models this situation is defined as follows:

y = mx + b.

In which the parameters are as follows:

m is the slope, which is the number of games that he wants to win per month.b = 2 is the intercept, which is the number of games that he has already won.

They want to win more than 57 games, and there are 5 months to play, hence the inequality is given as follows:

5m + 2 > 57.

It is solved similarly to an equality, isolating the variable m and finding the range of values, as follows:

5m > 55

m > 55/5

m > 11.

Hence the team needs to win more than 11 games a month, and the solutions is illustrated by the image given at the end of the answer.

More can be learned about linear functions at https://brainly.com/question/24808124

#SPJ1

f(x) = 4
this line is
vertical
O horizontal
O diagonal
O impossible to graph

Answers

f(x) = 4

this line is

O vertical

O horizontal

O diagonal

O impossible to graph

A model rocket is launched with an initial upward velocity of 113 ft/s. The rocket’s height h (in feet) after t seconds is giving by the following. h=113t-16t^2 Find all values of t for which the rockets height is 39 feet. Round your answer(s) to the nearest hundredth.

Answers

[tex]\begin{gathered} h=113t-16t^2 \\ h=39ft \\ 39=113t-16t^2 \\ 16t^2-113t+39=0 \\ Use\text{ quadratic formula to find t} \\ \frac{-b\pm\sqrt{b^2-4ac}}{2a} \\ a=16 \\ b=-113 \\ c=39 \\ \frac{-(-113)\pm\sqrt{-113^2-4\times16\times39}}{2\times16} \\ \frac{113\pm\sqrt{12769-2496}}{32} \\ \frac{113\pm\sqrt{10273}}{32} \\ \frac{113\pm101.355808911}{32} \\ \frac{113+101.355808911}{32}0r\text{ }\frac{113-101.355808911}{32} \\ \frac{214.355808911}{32}\text{ or }\frac{11.644191089}{32} \\ 0.36388097153\text{ or }6.69861902847 \\ t\approx0.36\text{ or 6.70} \end{gathered}[/tex]

In the diagram, .

Triangles G E F and J H I are shown. The length of side G F is 20 and the length of side I J is 10. Th elength of side F E is 40 and the length of side I H is 20.

To prove that the triangles are similar by the SAS similarity theorem, it needs to be proven that

Answers

In order prove that the triangles are similar by the SAS similarity theorem, it needs to be proven that A. J measures 60°.

What is SAS Similarity Theorem?

The SAS Similarity Theorem states that two triangles are similar if their included angles are congruent and their two sides are proportionate to one other. Triangle congruence and resemblance are both criteria for SAS.

The triangles are congruent if two sides and the included angle of one triangle are equivalent to two sides and the included angle of another triangle.

In this case, the length of side FE is 40 and the length of side IH is 20. This gives a value of 60. Therefore, J should measure 60°.

In conclusion, the correct option is A.

Learn more about triangles on:

https://brainly.com/question/21735282

#SPJ1

Triangles G E F and J H I are shown. The length of side G F is 20 and the length of side I J is 10. Th elength of side F E is 40 and the length of side I H is 20. To prove that the triangles are similar by the SAS similarity theorem, it needs to be proven that

J measures 60°.

J measures 30°.

I measures 60°.

I measures 30°.

What is the smallest 3 digit number that is divisable by 2, 3, 4 , 5 and 6

Answers

The  smallest 3 digit number that is divisible by 2, 3, 4 , 5 and 6 is 720.

How can the digit number that is divisible  be caculated?

Let us find the factors of the given numbers which are Factors of 2 are 1 and 2. then the factors of  3 are 1 and 3, the Factors of 4 are 1, 2 and 2, then the Factors of 5 are 12 and 5, then the Factors of 6 are 12, 2 and 3.

Then we can find the Lowest Common Factor of them as (1*2*3*2*2*5*2*3) which will give us 720.

Read more about digit number at:

https://brainly.com/question/15707831

#SPJ1

what is8.546 rounded to nearest tenth

Answers

If we round to the nearest tenth, it would be 8.5 because the hundredth digit is less than 5.

Hence, the answer is 8.5.

6. rule (x,y) ---> is the image similar to the original pre-image?

Answers

Here, we want to check for the relationship between the image and its pre-image

The pre-image is (x,y)

The image is (3x,3y+5)

As we can see, the pre-image is not similar

This is because the transformation applied to the two values are not same

Thus, we have that;

No, the image is not similar to the pre-image as the translations applied to both coordinates are not same

The pre-image was transformed by dilating the x-coordinate of the pre-image by 3 units while the y-coordinate was transformed by dilating the y-coordinate of the pre-image by 3 and translating it upward by 5 units

Joshua earned $646 for 40 hours. Find the unit rate.

Answers

To find the unit rate (amount per hour) you divide the given amount into the given number of hours:

[tex]\frac{646}{40h}=16.15/h[/tex]

Then, the unit rate is $16.15/hour

Translate to an equation, then solve.The product of 2, and a number increased by 7, is – 36.

Answers

Let the number be r then

2r is the product of 2 and the number

2r + 7 is when the product is increased by 7

2r + 7 = -36

2r = -36 - 7

2r = -41

r = -41/2

= -20 1/2

PLEASE HELP ITS DUE SOON! I DONT GET ANY OF THIS! HELP WOULD BE MUCH APPRECIATED! NEED THIS DONE BEEN STUCK ON THIS FOR WAY TO LONG!
YOU WILL GET 100 POINTS IF YOU HELP! QUESTION DOWN BELOW!!!!!

Answers

1) [tex]BC \parallel EF, \angle 1=\angle 3[/tex] (given)

2) [tex]\angle 2=\angle 3[/tex] (corresponding angles)

3) [tex]\angle 1=\angle 2[/tex] (transitive property)

4) [tex]AB \parallel DE[/tex] (converse of corresponding angles theorem)

Can someone please help me graph this? It’s due in 5 minutes

Answers

An order relationship with greater than, greater than or equal to, less than, or less than or equal to between two algebraic expressions. The solution of inequality -3x+4≥10 is x≤-2

What is Inequality?

An order relationship with greater than, greater than or equal to, less than, or less than or equal to between two algebraic expressions.

The given inequality is

-3x+4≥10.

To solve the above inequality  

Subtract -4 from both sides.

-3x+4-4≥10-4

+4 and -4 we get zero.

-3x≥6

Divide by 3 on both sides

-x≥2

When negative sign is multiplied on both sides, the greater than or equal to symbol changes to lesser than or equal to.

x≤-2

Hence x≤-2 is solution for -3x+4≥10.

To learn more on Inequality click:

https://brainly.com/question/20383699

#SPJ1

Other Questions
Discuss the impact of the Age of exploration on Africa, Asia, and Europe. in the tftp protocol: if the client changes its port number on a subsequent connection, but the server does not, what prevents an old-duplicate data packet sent by the server from being accepted by the new client? if the server changes its port number on a subsequent connection, but the client does not, what prevents an old-duplicate din an rpc-like protocol in which multiple requests can be outstanding, and replies can be sent in any order: assume that requests are numbered and that ack[n] acknowledges reply[n]. should acks be cumulative? if not, what should happen if an ack is lost?ata packet sent by the server from being accepted by the new client? What is the amount of valence electrons in an atom equivalent to?The amount of protons an atom hasThe group the atom is locatedThe period the atom is locatedThe amount of electrons an atom has Which of the following statements about visual aids is true?Visual aids are used to clarify meaning, emphasize main points, or help illustrate a speaker's message.Most visual aids should be handmade, and nothing like video or a computer PowerPoint presentation should be used.All of the choices are correct.Visual aids are very distracting to audiences and should only be used if the topic of the speech is very complicated or technical.Speakers should use as many visual aids as possible to help communicate their messages to audiences. 2.In the context of the text, how can we achieve peace? What does Louise Mushikiwabobelieve is necessary for Rwanda to move on from its history with genocide? Do you agreewith this course of action? Find and interpret the slope for the real-world situation. Enter the slope in simplest form. Type your response in the box.Think of an organism that you find interesting. Research the organism using credible websites. Then list onestructural, one behavioral, and one functional adaptation that the organism uses. help mee this is an ireadu question HELPPPPHow did the British view the United States after the American Revolution?A. They recognized the United States but kept their land in the South.B.They refused to recognize the United States or give them land, but they agreed to peace.C. They recognized the United States as its own nation and gave up British territory.D. They gave Americans land but refused to recognize the United States. On the bright side, the rain made the grass grow in the neighborhood. You like to mow the neighbor's lawns, and you charge $25 per lawn you mowed 6 lawns how much did u make mowing 6 lawns which of the following serves as the sensory relay center whose neurons project to both the amygdala and the higher cortical regions for further processing? question 3 options: 1) hippocampus 2) both the thalamus and the hippocampus 3) thalamus 4) hypothalamus Find the volume of the rectangular prism.8 cm8 cm3 cm Listen to the audio and choose the correct relationship between Paula and Roberto. A. Paula es la abuela de Roberto. B. Roberto es el primo de Paula. C. Paula es la prima de Roberto. D. Paula es la ta de Roberto. A group of friends wants to go to the amusement park. They have no more than $280 to spend on parking and admission. Parking is $20, and tickets cost $40 per person, the including tax. Write and solve an inequality which can be used to determine x, number of people who can go to the amusement park. WILL GIVW BRANLIEST DUE TODAY thank you>>>Three meals per day are provided by the hotel. The hotel will charge a total of $9,240 for all meals for the entire group. What is the total cost of meals for each person. rafe is doing research for his informative speech and has not been happy with the quality of information he's been finding. he goes to his university library, where he searches psycinfo, a(n) focused on the narrow discipline of psychology. Given: m//nProve: m2 + m28 = 180 since his cerebrovascular accident, a man has been denying his left hemiplegia. what term is used to describe this finding?since his cerebrovascular accident, a man has been denying his left hemiplegia. what term is used to describe this finding? Orlando makes a trail mix for his hiking trip. He uses 1 \frac{5}{6}1 6 5 pounds of cashews, 1.6 pounds of sunflower seeds, 1 \frac{1}{8}1 8 1 pounds of raisins, and 1.625 pounds of pecans. Which answer choice shows these weights in order from least to greatest ? DUE TODAY NEED NOW WILL GIVE BRAINLIEST what part of a supreme court decision presents the argument in opposition to the court's ruling